Sie sind auf Seite 1von 12

Exam FM/2

Practice Exam 3
c
Copyright 2013
Actuarial Investment.
1. An n-year bond has a face amount of 1000 with annual coupons of 5%. The book value of
the bond at the end of the 6th year is 857 and the adjustment to the bond in the 6th year is a
write-up of 11. Calculate n.
(A) 12
(B) 13
(C) 14
(D) 15
(E) 16
2. An investor buys a 10-year bond for X. The bond pays semiannual coupons at a rate of 5%
and is priced to yield 7.5% convertible semiannually. The investor reinvests all coupons into
an account earning interest at an annual effective interest rate of 6%. After 10 years, he has
5445. Calculate X.
(A) 2431
(B) 2579
(C) 2696
(D) 2703
(E) 2928
3. Which of the following financial instruments carries the highest credit risk?
(I) A forward contract purchased over-the-counter
(II) A futures contract
(III) A put option purchased over-the-counter
(A) (I)
(B) (II)
(C) (III)
(D) (I) and (II) both have the highest credit risk
(E) (I) and (III) both have the highest credit risk

4. A 4-year annuity makes payments at the beginning of every month starting today. The first
12 payments are 1000 each. The next 12 payments are 1100 each. The next 12 payments are
1210 each. The final 12 payments are 1331 each. What is the value of the annuity at the end
of 4 years valued at an interest rate of 12% convertible monthly?
(A) 43438
(B) 43872
(C) 70032
(D) 70732
(E) 89073
5. A company has a liability of 1000 due in 1.95 years. The company wants to provide Redington immunization for the liability by purchasing a combination of bond X and bond Y ,
which are available in any face amount. Bond X is a 2-year bond with annual coupons of
11%. Bond Y is a 2-year zero-coupon bond. The annual effective rate of interest is 5%.
What amount of bond Y should be purchased to provide Redington immunization for this
liability?
(A) 385
(B) 434
(C) 471
(D) 518
(E) 575
6. A 10-year bond with annual coupons of 5% is priced to yield an effective annual interest rate
of 8.3%. A 10-year bond with semiannual coupons at a rate of 5% has the same face amount
and price and yields an annual effective interest rate of i%. Calculate i.
(A) 8.25
(B) 8.30
(C) 8.42
(D) 8.51
(E) 8.57

7. Betty buys a 30-year bond with annual coupons of 4% priced to yield 7% for a purchase
price of X. Coupon payments from the bond are reinvested into an account earning interest
at an annual effective rate of 6%. After 30 years, Betty has 100,000. Calculate X.
(A) 14977
(B) 15081
(C) 15119
(D) 15228
(E) 15414
8. A portfolio worth 500 produces cashflows of 150 at time 3, X at time 4, and 200 at time
6. The cashflow of X at time 4 makes up 45% of the value of the portfolio. Calculate the
Macaulay duration of the portfolio.
(A) 3.67
(B) 4.00
(C) 4.33
(D) 5.00
(E) 5.50
9. Which of the following are true regarding swaps?
(I) A prepaid swap is a swap in which the present value of all payments, calculated using the
current term structure, is paid immediately
(II) A deferred swap is a swap in which the future value of all payments, calculated using
the current term structure, is paid at the end of the swaps term
(III) The swap spread is collected by an intermediary for the service of arranging a swap
between two parties
(A) (I) only
(B) (II) only
(C) (I) and (III)
(D) (II) and (III)
(E) The answer is not given by any of (A), (B), (C), or (D)

10. Two bonds have the same yield rate. Both bonds have a face amount of 1000, a redemption
amount of 1100, and a price of 800. The first is a 15-year bond with annual coupons at a
coupons rate of 5%. The second is a 20-year bond with coupons at a coupon rate of r%
convertible semiannually. Calculate r.
(A) 2.43
(B) 2.68
(C) 4.87
(D) 5.35
(E) 5.45
11. Which of the following are true concerning the cost of carry of a long position in a stock?
(I) The cost of carry includes any dividends paid by the stock.
(II) The cost of carry includes the full price at purchase of the stock.
(III) The cost of carry is zero if the stock pays no dividends and if the increase in stock price
exactly matches the annual effective rate of interest.
(A) (I) only
(B) (III) only
(C) (II) and (III)
(D) (I), (II), and (III)
(E) The answer is not given by any of (A), (B), (C), or (D)
12. Which of the following are true about convertible bonds?
(I) A convertible bond is always a zero-coupon bond.
(II) A convertible bond does not have a fixed maturity date.
(III) If a companys stock value falls, the value of a convertible bond issued by the company
also must fall.
(A) (I) only
(B) (III) only
(C) (I) and (II)
(D) (II) and (III)
(E) The answer is not given by any of (A), (B), (C), or (D)

13. Adam and Jane both invest the same amount of money into 10-year bonds that have annual
coupons and are priced to yield 6%.
Adams bond has coupons of 5%, and Adam reinvests coupons into an account earning
interest at an effective annual rate of 12%.
Janes bond has coupons of 15%, and Jane reinvests coupons into an account earning interest
at an effective annual rate of i%.
After 10 years, Adam and Jane have the same amount of money. Calculate i.
(A) 6.4
(B) 7.7
(C) 8.2
(D) 9.0
(E) 9.8
14. At time t = 0, Roger deposits $100 into an account earning interest at a force of interest of
1
. At time t = 4, he transfers all of the money to a second account earning interest at a
1+3t
rate of 12% compounded monthly. How much money does Roger have at time t = 8?
(A) 200
(B) 379
(C) 422
(D) 429
(E) 560
15. A 30-year bond has a face amount of 10,000 and a redemption amount of 12,000. It has
annual coupons of 5% and the purchase price is 10,000. Calculate the adjustment to book
value in the 5th year.
(A) Write-down of 103.87
(B) Write-down of 35.21
(C) No adjustment
(D) Write-up of 35.21
(E) Write-up of 103.87

16. Which of the following positions could result in unlimited gain?


(I) Short position in a call ratio
(II) Long position in a put option
(III) Short position in a strangle
(A) (I) only
(B) (III) only
(C) (I) and (II)
(D) (I), (II), and (III)
(E) The answer is not given by any of (A), (B), (C), or (D)
17. At an annual effective interest rate of 7%, the accumulated value of an annuity due with n
annual payments of 10 is 110. At an annual effective rate of i%, the accumulated value of
an annuity immediate with n annual payments of 13 is 140.
Calculate i.
(A) 5.1
(B) 5.8
(C) 6.5
(D) 7.3
(E) 8.3
18. Daniel writes a covered call with maturity in six months, a premium of 4, and a strike price
of 38. He also buys a put option for the same underlying asset with maturity in six months,
a premium of 4, and a strike price of 38. After six months, the underlying assets price is 44.
The annual effective interest rate is 11.8%. What is Daniels profit?
(A) -10
(B) 0
(C) 10
(D) 12
(E) 18

19. A stock costs 54 and will pay a dividends of 6 this year. The annual effective rate of interest
is 6%. Let X be the one-year cost of carry of a long position in the stock, Y be the one-year
cost of carry of a short position in the stock, and Z be the one-year cost of carry of a long
position in a futures contract in the stock. What is the relationship between X, Y , and Z?
(A) X > Y > Z
(B) X > Z > Y
(C) Y > X > Z
(D) Y > Z > X
(E) Z > X > Y
20. A loan with amount A makes n level annual payments, where n is even, beginning 1 year
after the loan is taken out. The annual effective interest rate is i such that 0 < i < 1. Which
of the following scenarios are possible?
(I) The interest paid in year

n
2

equals the principal paid in year n2 .

(II) The principal paid in year 1 equals the interest paid in year n.
(III) Immediately after payment n2 , the oustanding balance on the loan is 21 A.
(A) (I) only
(B) (II) only
(C) (I) and (III)
(D) (II) and (III)
(E) The answer is not given by any of (A), (B), (C), or (D)
21. A 10-year zero-coupon bond has a face value of 1000. Let i be the annual effective interest
rate such that i > 0, MacD be the bonds Macaulay duration, and ModD be the bonds
modified duration. Which of the following relationships is true?
(A) MacD < ModD < 10
(B) MacD < ModD = 10
(C) ModD < MacD < 10
(D) ModD < MacD = 10
(E) It cannot be shown that any of (A), (B), (C), or (D) is true.

22. It is known that K1 < K2 . An investor has purchased a combination of options that result in
a position equivalent to a K1 -K2 bear spread. What transaction will change his position to
be equivalent to a long position in a ratio spread?
(A) Buy a call option with strike price K1
(B) Buy a call option with strike price K2
(C) Write two call options with strike price K2
(D) Buy a put option with strike price K1
(E) The answer is not given by any of (A), (B), (C), or (D).
23. On January 1, a fund has a balance of 1000. At the end of each month for 12 months, interest
is credited to the account at a rate of 12% convertible monthly, and a deposit of 100 is made
into the account. Calculate the time-weighted rate of return of the fund.
(A) 12.0%
(B) 12.7%
(C) 17.3%
(D) 29.5%
(E) 39.5%
24. A zero-coupon bond has a modified duration of 8. Calculate the convexity of the zero-coupon
bond assuming an annual effective rate of interest of 12.5%.
(A) 56.9
(B) 64.0
(C) 71.1
(D) 85.3
(E) 91.1
25. A loan is to be repaid in n years, where n is divisible by 3, with level annual payments at
the end of each year. The principal paid during year 13 n is one-half the principal paid during
year 23 n. The annual effective interest rate is 8%. Calculate n.
(A) 15
(B) 18
(C) 21
(D) 24
(E) 27

26. The following chart shows prices for zero-coupon bonds with face amount 1000. It is known
that the one-year forward rate two years from now is 6.1%.
Time to maturity (years)
1
2
3

Zero-coupon bond price


930.00
X
X 50.26

Calculate X.
(A) 756.57
(B) 791.05
(C) 840.05
(D) 874.19
(E) 892.42
27. A company has a liability of 800 in 6 years. The company wants to provide Redington immunization for the liability by purchasing two zero-coupon bonds are available for purchase
at any face amount. Bond X matures in 4 years and bond Y matures in 8 years. The annual
effective rate of interest is 6%. What amount of bond X should be purchased to provide
Redington immunization for this liability?
(A) 255
(B) 276
(C) 309
(D) 327
(E) 356
28. The price of a 10-year bond, which is 100 less than its face amount, is equal to the present
value of the face amount. It is known that j = r + .03, where j is the annual yield rate and
r is the annual coupon rate. Calculate the face amount.
(A) 291
(B) 301
(C) 332
(D) 391
(E) 440

29. A company has a liability of 1000 due in 2.6 years. The company purchases a 3-year bond
with face amount F and annual coupons of r% to provide Redington immunization for the
liability. The annual effective rate of interest is 6%. Calculate r.
(A) 12.6
(B) 14.7
(C) 16.4
(D) 18.1
(E) 19.7
30. It is known that K1 < K2 . Which of the following positions create a bear spread?
(I) Buying a call with strike K1 and selling a call with strike K2
(II) Buying a call with strike K2 and selling a call with strike K1
(III) Selling a covered written call with strike K1 and selling a put with strike K2
(IV) Buying a put with strike K1 and selling a put with strike K2
(A) (I) only
(B) (I) and (IV)
(C) (II) only
(D) (II) and (III)
(E) The answer is not given by any of (A), (B), (C), or (D)
31. ABC Co. takes out a loan for 10,000. For the first 19 years, ABC Co. makes payments at the
end of each year of 120% of the interest due. At the end of the 20th year, ABC Co. makes a
final balloon payment of X.
The annual effective interest rate is 8%. Calculate X.
(A) 7243
(B) 7360
(C) 7497
(D) 7822
(E) 7949

10

32. A 12-year loan makes 11 annual payments of 100 at the end of each year followed by a
balloon payment of 400 at the end of year 12. The annual effective interest rate is i. Which
of the following represents the interest paid in the 4th payment?
1 9
)
(A) 100 100( 1+i
1 9
(B) 100 + 400( 1+i
)
1 9
1 8
) 400( 1+i
)
(C) 100( 1+i
1 8
1 9
(D) 300( 1+i
) 200( 1+i
)
1 9
1 8
) 400( 1+i
)
(E) 100 + 300( 1+i

33. A geometric annuity immediate with 24 monthly payments is worth 3000. A geometric
annuity immediate with 12 monthly payments is worth 1000. The first payment of both
annuities is the same, and the payments of the annuities increase at the same rate. The
annual effective rate of interest is 10%. What is the amount of the final payment of the
24-month annuity?
(A) 253.20
(B) 271.65
(C) 290.10
(D) 308.55
(E) 327.00
34. Order the following quantities from smallest to greatest valued at the same annual effective
interest rate.
I. The present value of an annuity immediate with 12 annual payments of P .
II. The present value of an annuity due with 12 annual payments of P .
III. The present value of an annuity immediate with 12 monthly payments of P .
IV. The present value of an annuity due with 12 monthly payments of P .
V. The present value of an 12-month continuously payable annuity paying a total of 12P
during the annuity.
(A) I, II, III, IV, V
(B) I, II, III, V, IV
(C) I, III, II, V, IV
(D) I, III, V, II, IV
(E) The answer is not given by any of (A), (B), (C), or (D).

11

35. An investor creates a 2:1 ratio spread by buying one call with strike price 105 and selling
two calls with strike price X. If the spot price at maturity were 130, the investors payoff
would be 5. What is the investors maximum possible payoff?
(A) 10
(B) 15
(C) 20
(D) 25
(E) 30

12

Das könnte Ihnen auch gefallen